Gradient With Respect to a Set of Coordinates

In summary, we have shown that the gradient of a scalar function ##U## with respect to the difference in positions between two particles is equal to the negative of the gradient of ##U## with respect to the first particle's position. This can be extended to multiple dimensions by using vector notation.
  • #1
cwill53
220
40
TL;DR Summary
Prove that ##\nabla_1U(\mathbf{r}_1- \mathbf{r}_2 )=-\nabla_2U(\mathbf{r}_1- \mathbf{r}_2 )##
In physics there is a notation ##\nabla_i U## to refer to the gradient of the scalar function ##U## with respect to the coordinates of the ##i##-th particle, or whatever the case may be.

A question asks me to prove that

$$\nabla_1U(\mathbf{r}_1- \mathbf{r}_2 )=-\nabla_2U(\mathbf{r}_1- \mathbf{r}_2 )$$

How do you actually extend this idea to ##\mathbb{R}^n## though? I'm not sure if everything I have written below makes sense or is correct, I'm hoping to get some clarification. The weakness is in my mathematics.

Suppose I have ##X## be an open set in ##(\mathbb{R}^n )^m## and ##f:X\rightarrow \mathbb{R}## a scalar valued function; and let ##A=\left \{ \mathbf{r}_1, \mathbf{r}_2, ..., \mathbf{r}_m \right \}\subseteq X##.

Then

$$\nabla f= \begin{bmatrix}
\frac{\partial f}{\partial x _{1_{1}}}& \frac{\partial f}{\partial x _{2_{1}}} &\cdots & \frac{\partial f}{\partial x _{n_{m}}}
\end{bmatrix}^T \in (\mathbb{R}^n )^m $$

$$\nabla_k f= \begin{bmatrix}
\frac{\partial f}{\partial x _{1_{k}}}& \frac{\partial f}{\partial x _{2_{k}}} &\cdots & \frac{\partial f}{\partial x _{n_{k}}}
\end{bmatrix}^T \in \mathbb{R}^n $$

where the subscript ##k## denotes that the derivatives are taken with respect to the $n$ coordinates of the ##k##-th element of the indexed set ##A=\left \{ \mathbf{r}_1, \mathbf{r}_2, ..., \mathbf{r}_m \right \}\subseteq X##. Let ##\mathbf{r}_i, \mathbf{r}_j \in A##, and let

$$
\mathbf{r}_i= \begin{bmatrix}
x_{1_{i}} & x_{2_{i}} & \cdots & x_{n_{i}}
\end{bmatrix}$$
$$\mathbf{r}_j= \begin{bmatrix}
x_{1_{j}} & x_{2_{j}} & \cdots & x_{n_{j}}
\end{bmatrix}
$$

and let ##\mathbf{x}= \mathbf{r}_i- \mathbf{r}_j##.

My questions are:

1. **Is it true that ##\nabla_k f \in \mathbb{R}^n## ?**
2. **Given the above, how should ##\nabla _i f(\mathbf{x})## be written? Can someone give explicit steps for the chain rule manipulations that need to occur?**
 
Physics news on Phys.org
  • #2
cwill53 said:
TL;DR Summary: Prove that ##\nabla_1U(\mathbf{r}_1- \mathbf{r}_2 )=-\nabla_2U(\mathbf{r}_1- \mathbf{r}_2 )##

A question asks me to prove that
In a simple one-dimension case of harmonic oscillator
[tex]U(x_1 - x_2) =\frac{k}{2} ( x_1 - x_2)^2 [/tex]
[tex]\frac{\partial U}{\partial x_1}=k(x_1-x_2)=-\frac{\partial U}{\partial x_2}[/tex]
Forces from the spring are same but have opposite signs at the ends.

Do you want to extend it from 1D to 3D or more higher dimensions, or from 2 to more ends or particles, or both ?
 
Last edited:
  • Like
Likes topsquark
  • #3
cwill53 said:
TL;DR Summary: Prove that ##\nabla_1U(\mathbf{r}_1- \mathbf{r}_2 )=-\nabla_2U(\mathbf{r}_1- \mathbf{r}_2 )##
I'm not sure I follow what you are doing there. If we have two particles in 3D, then we can represent the position of each particle as:$$\mathbf{r_1} = (x_1, y_1, z_1), \ \mathbf{r_2} = (x_2, y_2, z_2)$$Technically, the potential is a function of six coordinates:$$V:\mathbb R^6 \rightarrow \mathbb R$$Where the first three coordinates represent the position of first particle and the second three coordinates represent the second particle.

Assuming, however, that the potential is a function of the difference in position between the particles, then we have a "reduced" potential function:$$U: \mathbb R^3 \rightarrow \mathbb R$$Where the argument here is the difference in position of the two particles.

I've used two different letters here to emphasise the different functions involved. Technically, we have:
$$V(\mathbf{r_1}, \mathbf{r_2}) \equiv V(x_1, y_1, z_1, x_2, y_2, z_2) = U(\mathbf{r_1} - \mathbf{r_2}) \equiv U(x_1 - x_2, y_1 - y_2, z_1 - z_2)$$Now, we have two gradient functions defined here:
$$\mathbf{\nabla_1}V = (\frac{\partial V}{\partial x_1}, \frac{\partial V}{\partial y_1}, \frac{\partial V}{\partial z_1})$$$$\mathbf{\nabla_2}V = (\frac{\partial V}{\partial x_2}, \frac{\partial V}{\partial y_2}, \frac{\partial V}{\partial z_2})$$And, now we can see what is meant by these two gradients acting on the potential function ##U(\mathbf{r_1} - \mathbf{r_2})##:
$$\frac{\partial V(x_1, y_1, z_1,x_2, y_2, z_2)}{\partial x_1} = \frac{\partial U(x_1-x_2, y_1-y_2, z_1 - z_2)}{\partial x_1} = \frac{\partial U(x_1 - x_2, y_1 - y_2, z_1-z_2)}{\partial x}$$Where ##\frac{\partial U}{\partial x}## is just the usual partial derivative of ##U## with respect to its first argument (which we've called ##x##). And, of course, we have the equivalent for ##y_1## and ##z_1##.

And, to take the partial derivate wrt the second coordinates, we need to use the chain rule:
$$\frac{\partial V(x_1, y_1, z_1,x_2, y_2, z_2)}{\partial x_2} = \frac{\partial U(x_1-x_2, y_1-y_2, z_1 - z_2)}{\partial x_2} = -\frac{\partial U(x_1 - x_2, y_1 - y_2, z_1-z_2)}{\partial x}$$This leads us to the conclusion that:
$$\mathbf{\nabla_1}V = -\mathbf{\nabla_2}V$$In the special case where the potential can be written as a function ##U## of the difference in positions.

Finally, we can adopt the notation:$$\mathbf{\nabla_1}U(\mathbf{r_1} - \mathbf{r_2}) \equiv \mathbf{\nabla_1}V(\mathbf{r_1}, \mathbf{r_2})$$$$\mathbf{\nabla_2}U(\mathbf{r_1} - \mathbf{r_2}) \equiv \mathbf{\nabla_2}V(\mathbf{r_1}, \mathbf{r_2})$$To get the required result.

Note that a physicist would assume implicitly almost all of this supporting mathematical complexity. But, if you want to justify this fully, then this is what I've shown.

Finally, if you want to extend this to more dimensions, then the same steps apply at every stage, but the vector notation would get more complicated. Instead of ##x_1, y_1, z_1##, we would need something like ##\mathbf{r_1} = (x_{11}, x_{12} \dots x_{1n})## and ##\mathbf{r_2} = (x_{21}, x_{22} \dots x_{2n})## for the ##n## coordinates of the two particles.
 
  • #4
PS the simpler approach is just to assume/state that:
$$\frac{\partial U}{\partial x_1} = -\frac{\partial U}{\partial x_2}$$and hence$$\mathbf{\nabla_1}U = -\mathbf{\nabla_2}U$$Where ##U(\mathbf{r_1} - \mathbf{r_2})## is the potential function. Whether that constitutes a "proof" is a moot point.
 

1. What is a gradient with respect to a set of coordinates?

A gradient with respect to a set of coordinates is a mathematical concept used to describe the rate of change of a function in a particular direction. It is a vector that points in the direction of the steepest increase of the function at a given point.

2. How is the gradient with respect to a set of coordinates calculated?

The gradient with respect to a set of coordinates is calculated by taking the partial derivative of the function with respect to each coordinate and combining them into a vector. This vector represents the direction and magnitude of the gradient at a specific point.

3. What is the significance of the gradient with respect to a set of coordinates?

The gradient with respect to a set of coordinates is important in many fields of science and engineering as it allows us to understand and analyze the behavior of functions. It is used in optimization problems, vector calculus, and machine learning algorithms.

4. Can the gradient with respect to a set of coordinates be negative?

Yes, the gradient with respect to a set of coordinates can be negative. The sign of the gradient indicates the direction of the steepest increase of the function. A negative gradient means that the function is decreasing in that direction.

5. How does the gradient with respect to a set of coordinates relate to the concept of slope?

The gradient with respect to a set of coordinates is a generalization of the concept of slope. In one-dimensional space, the gradient is equivalent to the slope of a function. However, in higher dimensions, the gradient takes into account the rates of change in multiple directions, making it a more versatile and powerful tool in mathematical analysis.

Similar threads

Replies
3
Views
1K
  • Advanced Physics Homework Help
Replies
1
Views
918
Replies
2
Views
1K
  • Calculus
Replies
4
Views
1K
Replies
1
Views
4K
  • Advanced Physics Homework Help
Replies
5
Views
2K
  • Classical Physics
Replies
2
Views
889
Replies
4
Views
3K
Replies
18
Views
2K
Back
Top